32
$\begingroup$

Update: The negative answer to the following question has been provided by Matthew Daws, who won, but also rejected, the bounty of 100 euro that I set over the question.

Let $\mathcal M(\mathbb Z)$ be the set of all finitely additive probability measures on the power set of $\mathbb Z$. Let $\phi:\mathbb Z\rightarrow\mathbb R$ be nonnegative and bounded. Observe that $\phi$ is integrable with respect to any $\mu\in\mathcal M(\mathbb Z)$. Let me say that $\mu$ is $\phi$-translation invariant if for all $y\in\mathbb Z$ one has

$$ \int\phi(x+y)d\mu(x)=\int\phi(x)d\mu(x) $$

Let $I_\phi(\mathbb Z)$ be the class of $\phi$-invariant measures in $\mathcal M(\mathbb Z)$. Let $\mu\in I_\phi(\mathbb Z)$ be fixed.

Question: Is it true that the mapping $F:\nu\in\mathcal M(\mathbb Z)\rightarrow\int\int\phi(x+y)d\nu(x)d\mu(y)$ attains its maximum on a measure $\nu\in I_\phi(\mathbb Z)$?

Thanks in advance,

Valerio

$\endgroup$
10
  • 1
    $\begingroup$ Unfortunately Fubini's theorem does not hold. For intance, let $\phi=\chi_{\mathbb N}, $\mu$ a invariant measure such that $\int\phi d\mu=1$ e $\vu$ an invariant measure such that $\int\phi d\vu=0$. The lackness of Fubini's theorem is, at the end, the point. $\endgroup$ Sep 3, 2011 at 8:44
  • 1
    $\begingroup$ I think the title should be "Do invariant measures maximize the integral? (Bounty offered)" . That phrasing is slightly less crass, and stirs more curiosity; people will read through the post to find out what the bounty is and how much; I think the present phrasing of the title is in the grey area of acceptability on MathOverflow. Gerhard "It May Be Just Me" Paseman, 2011.11.18 $\endgroup$ Nov 18, 2011 at 17:27
  • 1
    $\begingroup$ @ Gerhard: After all, the late Paul Erdös also offered cash prizes for questions... (:-) $\endgroup$ Nov 18, 2011 at 18:47
  • 1
    $\begingroup$ Indeed he did Alain. And how many of those were phrased in a fashion appropriate for MathOverflow? I am not opposing the occasional practice; I just think the presentation should be improved. Gerhard "It Might Be Others Too" Paseman, 2011.11.18 $\endgroup$ Nov 18, 2011 at 18:58
  • 5
    $\begingroup$ There is a meta discussion, specifically discussing the bounty, at tea.mathoverflow.net/discussion/1212/…. My suggestion would be to explicitly spell out that the bounty is entirely independent of the usual mathematical conventions regarding acknowledgement and priority. $\endgroup$ Nov 18, 2011 at 21:28

1 Answer 1

23
$\begingroup$

Edit: Here is what I think is a counter-example.

Let $\phi$ be the indication function of the even natural numbers, let $\mathcal U$ be an ultrafilter supported on the even naturals, and let $\mathcal V$ be an ultrafilter defined on the even negative integers. Define $\mu\in M(\mathbb Z)$ by $$\int f(x) \ d\mu(x) = \lim_{x\in\mathcal V} f(x).$$ Then $\mu\in I_\phi$ (as, in fact, $\int \phi(x+y) \ d\mu(y)=0$ for all $x$). If $\nu\in I_\phi$ then $\nu$ must assign the same measure to $2\mathbb N$ and $2\mathbb N+1$, say $\alpha\leq 1/2$. You also need to argue that $\nu$ must assign zero measure to any finite set (else it won't be $\phi$-invariant). So for any $y\in\mathbb Z$, $$\int \phi(x+y) \ d\nu(x) = \nu(2\mathbb N-y) = \begin{cases} \nu(2\mathbb N) &: y\in 2\mathbb Z, \\ \nu(2\mathbb N+1) &: y\in 2\mathbb Z+1, \end{cases} = \alpha.$$ Thus $$\int \int \phi(x+y) \ d\nu(x) \ d\mu(y) = \int \alpha \ d\mu(y) = \alpha,$$ as $\mu$ is a probability measure. By contrast, let $\nu$ be defined by $$\int f(x) \ d\nu(x) = \lim_{y\in\mathcal U} f(y).$$ Then $$\int \phi(x+y) \ d\nu(x) = \begin{cases} 1 &: y\in 2\mathbb Z, \\ 0 &: y \in 2\mathbb Z+1, \end{cases}$$ and so $$\int \int \phi(x+y) \ d\nu(x) \ d\mu(y) = \int \chi_{2\mathbb Z}(y) \ d\mu(y) = 1.$$ So $F$ is not maximised on $I_\phi$.

In fact, by replacing $2\mathbb N$ by $k\mathbb N$, I think you get that $F$ has norm one, but $F(\nu)\leq 1/k$ for any $\nu\in I_\phi$.

But somehow, to my mind, what's wrong is that the $\mu\in I_\phi$ you choose is very poor. So here's a revised conjecture:

Let $\mu\in I_\phi$ maximise the integral $\int \phi(x) \ d\mu(x)$. Then $F$ attains its maximum on $I_\phi$.

Old post: (Explains my thinking).

I think of these questions using the Arens products, from abstract Banach algebra theory. So I work over the complex numbers; but this is not a problem.

Consider $A=\ell^1(\mathbb Z)$ with the convolution product, so $A$ is commutative. Then $A^*=\ell^\infty(\mathbb Z) = C(\beta\mathbb Z)$ is an $A$-module: $(a\cdot f)(b) = f(ba)$ for $a,b\in A,f\in A^*$. Then $A^{**}=M(\beta\mathbb Z)$ the space of finite Borel measures on the Stone-Cech compactification $\beta\mathbb Z$. Your space $M(\mathbb Z)$ is just the positive measures $\mu\in A^{**}$ with $\mu(1)=1$.

We try to extend the product of $A$ to $A^{**}$. Firstly we define a bilinear map $A^{**}\times A^*\rightarrow A^*$ by $$(\mu\cdot f)(a) = \mu(a\cdot f) \qquad (\mu\in A^{**}, f\in A^*, a\in A).$$ But then we have two choices for the product on $A^{**}$: $$(\mu \Box \lambda)(f) = \mu(\lambda\cdot f), \quad (\mu\diamond\lambda)(f) = \lambda(\mu\cdot f) \qquad (\mu,\lambda\in A^{**}, f\in A^*).$$ A little thought shows that $\mu\diamond\lambda = \lambda\Box\mu$.

So if $\phi\in A^*$ if positive then $\mu\in I_\phi$ if and only if $\mu\cdot\phi = \mu(\phi) 1$. This follows, as writing $\delta_x\in A=\ell^1(\mathbb Z)$ for the point mass at $x\in\mathbb Z$, we have $$(\phi\cdot\delta_x)(\delta_y) = \phi(\delta_{x+y}) \implies (\mu\cdot\phi)(\delta_x) = \mu(\phi\cdot\delta_x) = \int \phi(x+y) \ d\mu(y).$$ So the condition that $\mu\in I_\phi$ becomes that $(\mu\cdot\phi)(\delta_x)$ is constant in $x$, which is seen to be equivalent to $\mu\cdot\phi = \mu(\phi) 1$.

Similarly, your map $F$ is just $F(\nu) = (\mu\Box\nu)(\phi)$.

As you allude to, it's known that $\lambda\Box\mu \not= \mu\Box\lambda$ for arbitrary $\lambda,\mu$. However, we say that $f\in A^*$ is "weakly almost periodic" (WAP) if $(\lambda\Box\mu)(f) = (\mu\Box\lambda)(f)$ for all $\mu,\lambda\in A^{**}$. So if $\phi$ is WAP and $\mu\in I_\phi$ then for any $\nu\in M(\mathbb Z)$, $$F(\nu) = (\mu\Box\nu)(\phi) = (\nu\Box\mu)(\phi) = \nu(\mu\cdot\phi) = \nu(1) \mu(\phi) = \mu(\phi),$$ as $\nu$ is a probability measure. So actually $F$ is constant on $M(\mathbb Z)$ and so certainly attains its maximum at a point of $I_\phi$.

So, to be interesting, we need to ask the question for $\phi$ which are not WAP. An alternative characterisation of $\phi$ being in WAP is that the set of translates of $\phi$ in $\ell^\infty(\mathbb Z)$ forms a relatively weakly compact set. A nice characterisation of Grothendieck shows that this is equivalent to $$\lim_n \lim_m \phi(x_n+y_m) = \lim_m \lim_n \phi(x_n+y_m)$$ whenever all the limits exist, for sequences $(x_n),(y_m)$ in $\mathbb Z$. If $\phi$ is the indicator function of $\mathbb N$, then it's not in WAP.

We may as well assume that $\|\phi\|_\infty=1$. Another "easy" case is when we can find $\nu\in I_\phi$ with $\nu(\phi)=1$. Then $F(\nu) = \mu(\nu\cdot\phi) = \mu(1) \nu(\phi) = 1$; while for any $\lambda\in M(\mathbb Z)$, clearly $|F(\lambda)| = |\mu(\lambda\cdot\phi)| \leq 1$ as $\mu$ is a probability measure, and $\lambda\cdot\phi$ is bounded by $1$ (again, as $\lambda$ is a probability measure and $\phi$ is bounded by $1$). Notice that this case covers your example of when $\phi$ is the indicator function of $\mathbb N$.

So a test case is to find $\phi$ not in WAP and with $\nu(\phi)<\|\phi\|_\infty$ for all $\nu\in I_\phi$ (notice that $I_\phi$ is always non-empty, as $\mathbb Z$ is amenable). Do you have an example of such a $\phi$?

Actually, if $\phi$ is the indicator function of the even natural numbers, then that's an example. And that leads to my (hopeful) counter-example.

$\endgroup$
6
  • $\begingroup$ Matthew, I have just seen your answer. Give me some time to go through the details. Let me understand well: are you claiming that the answer is negative and give a counterexample in the Edit? $\endgroup$ Nov 18, 2011 at 19:10
  • $\begingroup$ Yes. Actually, let me make a final edit-- I'll make the counter-example clearer (I'll put it into your notation) and make a revised conjecture... $\endgroup$ Nov 18, 2011 at 20:30
  • $\begingroup$ I've some trouble to understand why that integral should give $\alpha$.. Indeed, my understanding is that $\int\int\phi(x+y)d\nu(x)d\mu(y)=\int_{2\mathbb Z}(\int\phi(x+y)d\nu(x))d\mu(y)+\int_{2\mathbb Z+1}(\int\phi(x+y)d\nu(x))d\mu(y)=$ $=\int\chi_{2\mathbb Z}d\nu(x)+\int\chi_{2\mathbb N+1}d\nu(x)=2\alpha$ Am I wrong? $\endgroup$ Nov 19, 2011 at 8:20
  • 1
    $\begingroup$ Hi Matthew, sorry for the delayed answer but I was mainly away this weekend. I am now sure that your counter-example is good. Please, contact me privately for the bounty. I am thinking about the missing property.. indeed, for my application, I have some stronger property and so there might be still a positive answer.. let me think about for a while. $\endgroup$ Nov 20, 2011 at 17:31
  • 39
    $\begingroup$ @Valerio: I'm glad the counter-example seems okay. I must say that I don't think I have done anything like enough work here to justify taking 100 euros off you (I hope that doesn't seem churlish). But what I will accept is, if you are ever in the North of England (or we meet at a conference), then you can buy me a drink or two... (and let us hope that still costs less than 100 euros after the current financial mess!) I'll email you shortly... $\endgroup$ Nov 20, 2011 at 20:07

Your Answer

By clicking “Post Your Answer”, you agree to our terms of service and acknowledge you have read our privacy policy.

Not the answer you're looking for? Browse other questions tagged or ask your own question.